1961 IMO Problems/Problem 4

Revision as of 20:16, 25 October 2007 by Temperal (talk | contribs) (IMO box)

Problem

In the interior of triangle $ABC$ a point $P$ is given. Let $Q_1,Q_2,Q_3$ be the intersections of $PP_1, PP_2,PP_3$ with the opposing edges of triangle $ABC$. Prove that among the ratios $\frac{PP_1}{PQ_1},\frac{PP_2}{PQ_2},\frac{PP_3}{PQ_3}$ there exists one not larger than $2$ and one not smaller than $2$.

Solution

This problem needs a solution. If you have a solution for it, please help us out by adding it.


1961 IMO (Problems) • Resources
Preceded by
Problem 3
1 2 3 4 5 6 Followed by
Problem 5
All IMO Problems and Solutions